Mathematical and Physical Journal
for High Schools
Issued by the MATFUND Foundation
Already signed up?
New to KöMaL?

Problem K. 479. (November 2015)

K. 479. In the expression \(\displaystyle \big({(-a^{-b})}^{-c}\big)^{-d}\) the numbers 1, 2, 3, 4 are substituted for \(\displaystyle a\), \(\displaystyle b\), \(\displaystyle c\), \(\displaystyle d\) in some order. In which case will the value of the expression be a maximum, and in which case will it be a minimum?

(6 pont)

Deadline expired on December 10, 2015.


Statistics:

124 students sent a solution.
6 points:Debreczeni Tibor, Földvári Ádám, Gilicze Márton, Hoffmann Balázs, Keltai Dóra, Kiss 468 Péter, Kluèka Vivien, Kovács 576 Kristóf, Mester Gyöngyvér, Pálvölgyi Szilveszter, Pinke Jakab Zoltán, Póta Balázs, Simon Dóra, Wirker Bálint.
5 points:44 students.
4 points:19 students.
3 points:13 students.
2 points:9 students.
1 point:11 students.
0 point:13 students.
Unfair, not evaluated:1 solutions.

Problems in Mathematics of KöMaL, November 2015